LSAT and Law School Admissions Forum

Get expert LSAT preparation and law school admissions advice from PowerScore Test Preparation.

User avatar
 Dave Killoran
PowerScore Staff
  • PowerScore Staff
  • Posts: 5852
  • Joined: Mar 25, 2011
|
#43451
Complete Question Explanation
(The complete setup for this game can be found here: lsat/viewtopic.php?t=16410)

The correct answer choice is (C)

If R visits Toronto, then of course she must be joined by H and either S or P. Since L then cannot visit Toronto, from the second rule she must visit Montreal, and she must be joined by either P or S (whoever is not travelling with the HR block).

The rule provided by this question stem therefore creates the following local diagram:
D92_Game_#1_#5_diagram 1.png
According to this diagram, answer choice (C) is the only choice that could be true.
You do not have the required permissions to view the files attached to this post.
 lathlee
  • Posts: 652
  • Joined: Apr 01, 2016
|
#39701
Hi. Actually even after reading answer analysis, I understand C and E would be the correct answer. But I do not get why C is correct over E.
 James Finch
PowerScore Staff
  • PowerScore Staff
  • Posts: 943
  • Joined: Sep 06, 2017
|
#39854
Hi Lathlee,

The way the game works is that ultimately we have one distribution, 3-2-0, with two templates, one where H-R-P visit Vancouver and S-L visit M or T:

  • M: (S/L)
    T: (L/S)
    V: H-R-P
And one template where V is the empty city and we have either H-R-S or H-R-P visit M or T, and L-S or L-P visit the other city. So:

  • M: H-R-(S/P)
    T: L-(P/S)
    V: (Empty)
and:

  • M: L-(S/P)
    T: H-R-(P/S)
    V: (Empty)
Question 5 says that R is in T, which puts us in the last template. As we can see, V can't have anyone travel to it, and P or S could visit either T or M.

This is a Could be True question, so always look first for the variables that can be in two or more spots, in this case S and P. Neither can be in V, but either one could be in M or T; (C) gives us P in T, and is thus correct.

Hope this helps!
 lathlee
  • Posts: 652
  • Joined: Apr 01, 2016
|
#39878
This clears up a lot . It just I got the wrong impression each city must have 2 visitors. I read your explanation and realized I made the wrong assumption. thx
 kupwarriors9
  • Posts: 73
  • Joined: Jul 01, 2021
|
#88428
I keep getting EVERY justify the conclusion type question wrong!!! This is the FIRST time I've struggled with something on logic games. Can someone give tips to tackling this type of question? Thanks

E.g. for example could you explain your methods in how to tackle this question?:

Removed due to LSAC copyrights. qeustion references: PT18, December 1992, LR2 (Section IV), #5, "The ends of modern centuries have been greeted with both apocalyptic anxieties and utopian fantasies..."
User avatar
 Dave Killoran
PowerScore Staff
  • PowerScore Staff
  • Posts: 5852
  • Joined: Mar 25, 2011
|
#88432
Hi Kup,

Every LSAT question has a specific post on this forum, in this case your question is linked to: viewtopic.php?f=690&t=5931

We've moved it over there, and the explanation already here above might be of help to you. Please let us know.

Also, per LSAC rules and copyrights no one cannot post the full text of LSAT questions, so I've removed that in your post and replaced it with a reference to the question.

All that aside, the good news is that Justify questions have some very powerful techniques that can be used to solve them. If you are a PowerScore course student or have the Logical Reasoning Bible I can refer you to places to lean more about those.

Thanks!
User avatar
 christinecwt
  • Posts: 74
  • Joined: May 09, 2022
|
#95324
Hi All - can anyone please help explain why Answer A is incorrect. Given that Lori can visit Toronto as well..... Thanks!
 Adam Tyson
PowerScore Staff
  • PowerScore Staff
  • Posts: 5153
  • Joined: Apr 14, 2011
|
#95326
The local restriction in this question puts R in Toronto, christinecwt, and R can never be in the same group as L. As the setup shows, H and R will always be in a group of 3 together with either S or P, and the other one of those two will be paired with L somewhere else. Thus, when R is in Toronto, Toronto has H, R, and either S or P. Since L can never go to Vancouver, she must go to Montreal with either P or S.
User avatar
 christinecwt
  • Posts: 74
  • Joined: May 09, 2022
|
#95327
Thanks a lot Adam! This solve my question! Many thanks!

Get the most out of your LSAT Prep Plus subscription.

Analyze and track your performance with our Testing and Analytics Package.